URGENT PLEASE HELP PLEASE

URGENT PLEASE HELP PLEASE

Answers

Answer 1

Answer: I believe the answer is A.

Explanation: hope this helps


Related Questions

The vertices of her garden are located at the points (−1,5), (4,2) and (9,−4) on a coordinate grid. If each unit on the grid represents a foot and the material costs $8 per foot, how much will she pay for the material on the side between points (−1,5) and (4,2)?

Answers

Given:

The vertices of the garden on a coordinate grid are (−1,5), (4,2) and (9,−4).

Each unit on the grid represents a foot and the material costs $8 per foot.

To find:

The cost for the material on the side between points (−1,5) and (4,2).

Solution:

Distance formula:

[tex]d=\sqrt{(x_2-x_1)^2+(y_2-y_1)^2}[/tex]

Using the above formula, the distance between points (−1,5) and (4,2) is

[tex]d=\sqrt{(4-(-1))^2+(2-5)^2}[/tex]

[tex]d=\sqrt{(4+1)^2+(-3)^2}[/tex]

[tex]d=\sqrt{(5)^2+(-3)^2}[/tex]

On further simplification, we get

[tex]d=\sqrt{25+9}[/tex]

[tex]d=\sqrt{34}[/tex]

[tex]d\approx 5.83[/tex]

Now,

1 unit = 1 foot and 1 foot material costs is $8.

So, 1 unit material cost is $8.

Cost of material for 5.83 units is

[tex]5.83\times 8=46.64[/tex]

Therefore, the cost for the material on the side between points (−1,5) and (4,2) is $46.64.

Solve -5 = 3x - 14.
X=__

Answers

Answer:

x=3

Step-by-step explanation:

-5 = 3x - 14

+14 + 14

3/9 = 3/3x

3 = x

Which of these graphs is the solution set for the equation y = -4?

Answers

Answer:A

Step-by-step explanation:

You are trying to find the Y-Intercept. So because there is no slope it is the Y-int.Also hope this is correct, sorry if it is wrong.

In a newspaper, it was reported that the number of yearly robberies in Springfield in 2012 was 90, and then went down by 50% in 2013. How many robberies were there in Springfield in 2013?​

Answers

Answer: 45

Step-by-step explanation:

John's dog slept for 3 hours. If the dog snored every 13 1 3 13 1 3 of an hour, how many times did he snore? Play media comment. Group of answer choices 9 times 1 time 12 times

Answers

Answer:

9 times

Step-by-step explanation:

A dog snores 1/3 of an hour

60 minutes = 1 hour

Hence,

1/3 of 60 minutes = 20 minutes

Hence, every 20 minutes, the dog snores

This means in 1 hour, the dog snores 3 times

The dog slept for 3 hours

Hence,

1 hour = 3 times

3 hours = x

Cross. Multiply

x = 3 × 3 times

x = 9 times

The dog snores 9 times

3. Given A(3,5), B(7, 10), C(0, 2), and D(1, a),
determine the value of a for which:
a) Line AB is parallel to line CD.
b) Line AB is perpendicular to line CD.
please help!! i’m failing grade 10 precal and i need this❤️ will give 15 points

Answers

Answer:

1)  if Line AB is parallel to line CD then value of a is: a=3

2) if Line AB is perpendicular to line CD then value of a is: a=1

Step-by-step explanation:

We are given A(3,5), B(7, 10), C(0, 2), and D(1, a) we need to find value of a for which:

We can use slope formula: [tex]Slope=\frac{y_2-y_1}{x_2-x_1}[/tex] to find value of a according to conditions given.

We are given:

[tex]x_1=3, y_1=5, x_2=7, y_2=10 \ for \ line \ AB \ and \\\x_1=0, y_1=2, x_2=1, y_2=a \ for \ line \ CD \[/tex]

a) Line AB is parallel to line CD.

When 2 lines are parallel there slope is same so, using this we can find value of a

[tex]Slope \ of \ line \ AB = Slope \ of \ line \ CD[/tex]

[tex]\frac{y_2-y_1}{x_2-x_1}= \frac{y_2-y_1}{x_2-x_1}\\\frac{10-5}{7-3}=\frac{a-2}{1-0} \\\frac{5}{5}=\frac{a-2}{1}\\1=a-2\\a=1+2\\a=3[/tex]

So, if Line AB is parallel to line CD then value of a is: a=3

b) Line AB is perpendicular to line CD.

When 2 lines are perpendicular there slopes are opposite of each other so, using this we can find value of a

[tex]Slope \ of \ line \ AB =-\frac{1}{Slope \ of \ line \ CD}[/tex]

[tex]\frac{y_2-y_1}{x_2-x_1}=-\frac{1}{ \frac{y_2-y_1}{x_2-x_1}} \\\frac{y_2-y_1}{x_2-x_1}=-\frac{x_2-x_1}{y_2-y_1}\\\frac{10-5}{7-3}=-\frac{1-0}{a-2} \\\frac{5}{5}=-\frac{1}{a-2}\\1=-\frac{1}{a-2}\\a-2=-1\\a=-1+2\\a=1[/tex]

So, if Line AB is perpendicular to line CD then value of a is: a=1

8. If k = log2 3 then log2 48 =

Answers

Answer:

16k

Step-by-step explanation:

k=3log2

k=(48\16) log2

16k= 48log2

16k=log2 48

Numbers can be expressed in logarithms.

The value of [tex]\mathbf{log_248}[/tex] is [tex]\mathbf{k + 4}[/tex]

The expressions are given as:

[tex]\mathbf{k = log_23}[/tex]

[tex]\mathbf{log_248}[/tex]

Express 48 as 3 * 16

[tex]\mathbf{log_248 = log_2(3 \times 16)}[/tex]

Split

[tex]\mathbf{log_248 = log_2(3) + log_2(16)}[/tex]

Express 16 as 2^4

[tex]\mathbf{log_248 = log_2(3) + log_2(2^4)}[/tex]

Rewrite as:

[tex]\mathbf{log_248 = log_2(3) + 4log_22}[/tex]

In logarithms:

[tex]\mathbf{log_aa = 1}[/tex]

So, we have:

[tex]\mathbf{log_248 = log_2(3) + 4(1)}[/tex]

[tex]\mathbf{log_248 = log_2(3) + 4}[/tex]

Substitute [tex]\mathbf{k = log_23}[/tex]

[tex]\mathbf{log_248 = k + 4}[/tex]

Hence, the value of [tex]\mathbf{log_248}[/tex] is [tex]\mathbf{k + 4}[/tex]

Read more about logarithms at:

https://brainly.com/question/8657113

Cheyenne is deciding between the ways she has to buy music. Albums on CD cost
$13 but downloads of an album only cost $10. If she can spend up to $150 on
music purchases, which of these expresses her purchasing restrictions when she
buys c CDs and downloads d albums?

Answers

Answer:

13c+10d>150

Step-by-step explanation:

Choose all the expressions that are equal to 34 × 8,010.
A. 8,010 × 34
B. 34 × (801 × 102)
C. 30 × 4 × 8,010
D. 34 × (8,000 + 10)
E. (30 + 4) × 8,010

Answers

Answer: A. 8,010 × 34

D. 34 × (8,000 + 10)

E. (30 + 4) × 8,010

Step-by-step explanation:

First we need to calculate 34 × 8010 = 272340

A. 8,010 × 34 = 272340

This is equal to 34 × 8010.

B. 34 × (801 × 102)

= 34 × 81702

= 2777868

This is not equal to 34 × 8010.

C. 30 × 4 × 8,010

= 961200

This is not equal to 34 × 8010.

D. 34 × (8,000 + 10)

= 34 × 8010 = 272340

This is equal to 34 × 8010.

E. (30 + 4) × 8,010

= 34 × 8010 = 272340

This is equal to 34 × 8010.

Therefore, the correct options are:

A, D, E.

Mike had 180 blue and red pencils. He had 45 blue. What percentage of his pencils are red.

Answers

Answer:

First divide 180 by 45, 180/45=4. So 25% of his pencils are blue, which means 75% of his pencils are red.

The percentage of red pencils Mike had is 75%.

What is the percentage?

A percentage is a value per hundredth. Percentages can be converted into decimals and fractions by dividing the percentage value by a hundred.

Given, Mike had 180 blue and red pencils and 45 were blue.

So, The number of red pencils is,

= (180 - 45).

= 135.

Therefore, The percentage of red pencils is,

= (135/180)×100%.

= 75%.

Some other concept related to percentage is percentage change,  

The net amount change will now be divided by the original sum and multiplied by 100%, giving us the percentage change, which is,

learn more about percentages here :

https://brainly.com/question/24159063

#SPJ2

Determine how the triangles are similar​

Answers

Answer:

SAS.. Can I have brainliest?

Step-by-step explanation:

Please help on a maths question, Brainiest for the best answer!!!

Answers

perimeter = sum of all sides

= 4.3 + 7 + 4.3 + 7 + 4.3 + 7 + 2.7

= 36.6 cm

Answer:

36.6 cm

Step-by-step explanation:

Each side, starting with the one all the way to the left and going anticlockwise.

7 + 4.3 + 7 + 4.3 + 7 + (7 - 4.3 = 2.7) + 4.3 = 36.6

In the activity, Heavy Flying, we looked at constraints for transporting packages from Careful Calculators and Charlie's
Chicken Feed
• Charlie's Chicken Feed containers weigh 40 pounds and are 2 cubic feet in volume.
• Careful Calculators containers weigh 50 pounds and are 3 cubic feet in volume.
On Monday, the pilot transferred containers that totaled 30,000 pounds and 1,600 cubic feet in volume.
1. Write a system of equations to model this situation.
2. Write a matrix equation to model this situation.
3. Solve. How many of each container was transported?
Show your work by uploading a picture or typing in the text box.

Answers

Answer: 500 chicken feed and 200 careful calculators

Step-by-step explanation:

2x+3y=1,600

40x+50y=30,000

those are your first two equations

your first matrix equation for A is 2, 3,40,50 and B is 1,600 and 30,000 when you enter those in your calculator and 500 and 200 should be your result.

a) The system of equations are 2x + 3y = 1600 and 40x + 50y = 30000

b) The matrix equations are solved

c) The solution to the system of equations is

The number of chicken feed = 500

The number of careful calculators = 200

What is multiplication of matrices?

The product of two matrices A and B is defined if the number of columns of A is equal to the number of rows of B. The first matrix must have the same number of columns as the second matrix has rows. The number of rows of the resulting matrix equals the number of rows of the first matrix, and the number of columns of the resulting matrix equals the number of columns of the second matrix

Given data ,

Let the number of chicken feed be represented as x

Let the number of careful calculators be y

Now , the equation will be

Charlie's Chicken Feed containers weigh 40 pounds and are 2 cubic feet in volume

And , Careful Calculators containers weigh 50 pounds and are 3 cubic feet in volume

Substituting the values in the matrices , we get

[tex]\left[\begin{array}{ccc}2 &3 \\ 40 &50 \\\end{array}\right] * \left[\begin{array}{ccc}x\\y\\\end{array}\right]= \left[\begin{array}{ccc}1600\\30000\\\end{array}\right][/tex]

On simplifying the matrix equation , we get

2x + 3y = 1600   be equation (1)

40x + 50y = 30000   be equation (2)

Now , multiply equation (1) by 20 , we get

40x + 60y = 32000   be equation (3)

Subtracting equation (2) from equation (3) , we get

10y = 2000

Divide by 10 on both sides of the equation , we get

y = 200 careful calculators

Substituting the value of y in equation (1) , we get

2x + 3 ( 200 ) = 1600

2x + 600 = 1600

Subtracting 600 on both sides of the equation , we get

2x = 1000

Divide by 2 on both sides of the equation , we get

x = 500 chicken feeds

Therefore , the value of x and y are 500 and 200 respectively

Hence , the system of matrix equations are solved

To learn more about matrix multiplication click :

https://brainly.com/question/13198061

#SPJ2

If f(x) = 2x2
+3 and g(x) = x2 - 7, find (f + g)(x).
A. x² - 4
B. X2 + 10
C. 3x2 - 4
D. 3x2 - 10

Answers

Answer:

Im stupid so dont trust me but imma guess and say c

Answer:

YEH its B X to the second power Plus 10

Step-by-step explanation:

There were 20 cups of pudding left in the cafeteria after lunch. The lunch aide poured the extra pudding into quart sized containers how many quart sized containers will she need?

Answers

Answer:

5

Step-by-step explanation:

1 galon =16 tazas

1/4 de galon=4 tazas

1+.25= 1.25 ó 1+1/4=1, 1/4 y eso lo multiplico por 4

1.25 ×4= 5

What is the slope of the line that intersects this two points (12,2), (-7,5)

Answers

(12,2) (-7,5)

x1 y1. x2 y2

use this formula to solve:

y2-y1/x2-x1

=

(5)-(2)

(-7)-(12)

slope: -3/19

Jesse plays basketball for his high school team. In his last 12 twelve games he scored the following points: 18, 22, 19, 15, 24, 22, 18, 15, 10, 14, 18, 10. Barbara plays basketball for her high school team. In her last 10 games she scored the following points: 15, 12, 12, 16, 18, 22, 16, 20, 20, 14. Determine which player had the largest range of points scored? What is the difference is the range of points between the two players?

In the form of a paragraph, explain in complete sentences the steps necessary to compute the range of the points, which team player had the largest range of points and include the final answer in your explanation. Complete your work in the space provided.

Answers

Answer:

Jesse has the largest range between the two players. The difference between their ranges is 2.

Step-by-step explanation:

Jesse's range: 14

Barbara's range: 10

Range is determined by using the largest number and the smallest number in a set of integers and subtracting them. Jesse's highest number was 24, and his lowest would be 10. 24-10=14.

Barbara's highest score was 22, while her lowest was 12. 22-12=10.

subtracting Jesse's range from Barbara's range would equal 2.  

(please don't cook me if this is incorrect but i'm 97% sure i'm right)

2x+7y =3
x= -4y

x=
y=

Answers

Answer:

Y=2

x=7

but they all equal 3

Y = -3
X = 12

Substitute the second equation into the first to get y and then using that you can work out x :)

PLEASEEEEEE HELPPPPP ASAPPPP

Answers

The correct answer is f(x)= 25z + 400

Plzz help me (sovle all question in the image)

Answers

9 is congruent by the lines on each side which match to be congruent lines

2.
Which of these expressions is equivalent to -2(x - 5)?
A.
-2x – 5
B.
- 2x + 5
C.
-2x + 10
-2x – 10
D.

Answers

Answer:

C.

Step-by-step explanation:

Evalute -2(x-5)

-2x+10

Answer:

-2x+10

Step-by-step explanation:

-2(x-5)

= -2x +10 { multiplying equation by -2}

Note:

(-)×(-) = (+)

(+) × (+) = (+)

(-) × (+) = (-)

(+) × (-) = (-)

A firefighter needs to lean a ladder against a wall of a building to reach a window. How far away from the base of the building should he/she place the foot of the ladder? Ladder is 25 feet long, window is 23.8 feet high in the building.

Answers

Answer:

7.7 feet

Step-by-step explanation:

Let the distance from the base of the building to the foot of the ladder be represented by x. Ladder is 25 feet long, while window is 23.8 feet high.

The length of the ladder, the height of the window and the distance from base of building to foot of ladder describes a right angled triangle.

Thus, applying the Pythagoras theorem;

[tex](25)^{2}[/tex] = [tex](23.8)^{2}[/tex] + [tex]x^{2}[/tex]

625 = 566.44 + [tex]x^{2}[/tex]

625 - 566.44 = [tex]x^{2}[/tex]

58.56 = [tex]x^{2}[/tex]

⇒ x = [tex]\sqrt{58.56}[/tex]

      = 7.653

The foot of the ladder should be placed 7.7 feet away from the base of the building.

Please help! this assignment is due under 10 minutes! please with explatiation Thank you!

Answers

can't see the question its blurry

Answer:

The 1st graph is The answer

Which set of data contains two outliers?
113, 115, 103, 114, 109, 111, 119
141, 151, 138, 142, 149, 140, 150
99, 113, 91, 104, 109, 114, 97
101, 135, 131, 99, 138, 136, 140

Answers

Answer:

D: 101, 135, 131, 99, 138, 136, 140

Step-by-step explanation:

edg2020 (do not get this confused with the question: "which set contains no outliers"

D

I got it right on the test

Need help with this
(I think it’s 49 but I’m not sure)

Answers

Answer:

Its 49

Step-by-step explanation:

Can i get brainliest

The perimeter of a rectangle 220 cm and its breadth and length are in
the ratio 4:7.What is the breadth and length?

Answers

Answer:

80 is the breadth and 140 is the length

Step-by-step explanation:

A car rental agency advertised renting a car for $23.95 per day and $0.24 per mile. If Shawn rents this car for 3 days, how many whole miles can he drive on a $150 budget?

Answers

Answer:

23.95(3)+ 0.24=150 multipy 3 and 23.95

71.85+0.24m=150 subtract 71.85 from 150

0.24m=78.15 divide 78.15 by 0.24

m=325 miles

Can someone help me? Giving 23 Points.

Answers

Answer:

Option 2 you welcome

explanation: I had this question a while back and I'm 100% sure that this is the right answer!

Writing an Equation Given Two Point
A line has an x-intercept of 3 and a y-intercept of -6.
The slope-intercept form of its equation is

Answers

The y-intercept is the value when x=0. The x-intercept of 3 corresponds to y=0, and the point on the line is (3,0). The y-intercept of 4 corresponds to x=0, and the point on the line is (0,4). ... Now that we have the slope, we can write the equation for the line in point-slope form. One quick way to do this is to use the fact that
3

4
=
12
to say an equation for this line is
4
x
+
3
y
=
12
(since, when
y
=
0
, it follows that
x
=
3
and when
x
=
0
it follows that
y
=
4
).
You could also find the slope of the line containing the points
(
x
,
y
)
=
(
3
,
0
)
and
(
x
,
y
)
=
(
0
,
4
)
as
Δ
y
Δ
x
=
4

0
0

3
=

4
3
so that the equation has the form
y
=

4
3
x
+
b
, where
b
is the
y
-intercept so the equation becomes
y
=

4
3
x
+
4
. This is equivalent to the last answer because you can multiply everything by 3 and rearrange to get
4
x
+
3
y
=
12.

Find the slope of the line that passes through (24,-5) and (87, 64).

Simplify your answer and write it as a proper fraction, improper fraction, or integer.

Answers

Answer:

Distance

80.7527089328897

Midpoint

(81.5,24)

Slope

7.27272727272727

x intercept

78.20

y intercept

−568.73

Step-by-step explanation:

Answer:

[tex]1 \frac{2}{21} [/tex]

Step-by-step explanation:

Please see the attached picture for the full solution.

Other Questions
1. Adding decimals is very similar to adding___________.2.Line up the number vertically so that all the decimal points are_______.3. Add extra______to the right of the number so that each number has the same number of digits to the right of the decimal point.4. Place the______________of the result in line with the other decimal points. What are the ramifications (pros and cons of introducing a natural predator of cane toads? Photosynthesis occurs inside of which organelle? a chloroplasts b the cell wall c mitochondria d ribosomes How do healthy cell decide when to divide? please help me, i dont like this Help please I will give Brainliest!!!!Josiah invested $96,000 in an account paying an interest rate of 3.5% compounded continuously. Assuming no deposits or withdrawals are made, how long would it take, to the nearest year, for the value of the account to reach $153,400? 40.5g of sugar is needed to make 5 cakes. How much sugar is needed for 8 cakes? Why is GNI per capita a better measure of standard of living than GDP? A. GNI per capita reflects all of the income within the economy. B. GNI per capita indicates how much each person makes on average. C. GNI per capita represents the value of goods and services in the economy. D. GNI per capita reflects the total market value of goods and services in the economy. i neeed answer fast thanks branches of sicence how does tom and jerry movie character influence your attitude How did the 17 amendment Change the way senators are elected Please select the word in parentheses that fits the sentence:A catalyst [weakens/strengthens] the bonds of the reactants [increasing/decreasing] the activation energy. Can someone help me with this??!!Explain the difference between -34 and (-3)4, and evaluate each power. can someone help me with these? Can anyone help me with this question of m I need Help with this Question Plzz!!! If f(x) = 2x^2 +1, complete the following: a. f(-4) b. f(-2)c. f(0) d. f(2) e. f(4) This is a timed test please i need help!! The following are budgeted data: January February March Sales in units 15,000 20,000 18,000 Production in units 18,000 19,000 16,000 One pound of material is required for each finished unit. The inventory of materials at the end of each month should equal 20% of the following month's production needs. Purchases of raw materials for February would be budgeted to be:______.A) 20,400 pounds.B) 19,600 pounds.C) 18,600 pounds.D) 18,400 pounds. A diver is on a board 1.80 m abovethe water. She jumps straight upat 3.62 m/s.At what speed does she hit the water?[?] m/s